0 Daumen
267 Aufrufe

Aufgabe:

Beweisen mithilfe vollständiger Induktion:

∀n∈ℕ gilt:

A(n): (1+\( \frac{2}{1} \)) *(1+\( \frac{2}{2} \)) *---*(1+\( \frac{2}{n} \)) =1+2...+n+(n+1)

Problem/Ansatz:

Ich habe im Induktionsanfang mit n=1 nachgewiesen, dass es sich um eine wahre Aussage handelt.

Für die Induktionsbehauptung habe ich A(n+1): (1+\( \frac{2}{n+1} \) )=(n+1)+(n+2)

Mein Problem liegt nun im Induktionsschritt, ich habe meine Probleme immer darin einen Anfang zu finden und würde mich über einen Tipp freuen.


LG Alena

Avatar von

1 Antwort

0 Daumen
 
Beste Antwort

Hallo,

Voraussetzung hast Du ja:$$A(n)=\prod\limits_{k=1}^{n}\left(1+\frac{2}{k}\right) = \sum\limits_{k=1}^{n+1}k \\ n=1:\quad \left(1+\frac{2}{1}\right) = 3 = 1+2 \space \checkmark$$

Mein Problem liegt nun im Induktionsschritt, ich habe meine Probleme immer darin einen Anfang zu finden

Ich setze mal voraus, dass die Gauß'sche Summenforrmel genutzt werden darf. Die da lautet:$$\sum\limits_{k=1}^{n+1}k = \frac{1}{2}(n+1)(n+2)$$Dann wird's relativ einfach:$$\begin{aligned} \prod\limits_{k=1}^{n+1}\left(1+\frac{2}{k}\right) &= \prod\limits_{k=1}^{n}\left(1+\frac{2}{k}\right) \cdot \left(1 +\frac{2}{n+1}\right) \\ &= \prod\limits_{k=1}^{n}\left(1+\frac{2}{k}\right) \cdot \left(\frac{n+3}{n+1}\right) \\ &= \frac{1}{2}(n+1)(n+2) \left(\frac{n+3}{n+1}\right) \\ &= \frac{1}{2}(n+2)(n+3)\\&= \sum\limits_{k=1}^{n+2} k\\&\text{q.e.d}\end{aligned}$$

Avatar von 48 k

Ein anderes Problem?

Stell deine Frage

Willkommen bei der Mathelounge! Stell deine Frage einfach und kostenlos

x
Made by a lovely community